3
$\begingroup$

I've seen this statement of Radstrom cancellation: if $A +C \subset B+C$ where $A,B$ are convex, $B$ is closed, and $C$ is bounded, then $A \subset B.$

Is it essential that $A$ be convex?

$\endgroup$
1
  • 4
    $\begingroup$ For those who are on first-read as confused as I was, A,B,C are subsets of a given normed vector space, and I assume that + is the addition of sets. $\endgroup$ Dec 15, 2009 at 5:32

3 Answers 3

9
$\begingroup$

No. Actually, the original version of the lemma does not require the convexity of A.

Reference: Hans Rådström (note spelling): An embedding theorem for spaces of convex sets. Proc. Amer. Math. Soc. 3 (1952) 165–169, MR0045938. doi:10.2307/2032477.

$\endgroup$
5
$\begingroup$

Alex, your idea of using a separation theorem can be turned into a full proof as follows:

As you said, $A \not\subset B$ iff there is a $a \in A$ and vector $x$ such that $\sup_{b \in B} \langle b,x \rangle < \langle a,x \rangle$. But then,

\begin{align} \sup_{z \in B + C} \langle z, x \rangle &= \sup_{b \in B, c \in C} (\langle b, x \rangle + \langle c, x \rangle ) \\\\ &= \sup_{b \in B} \langle b, x \rangle + \sup_{c \in C} \langle c, x \rangle \\\\ &< \langle a, x \rangle + \sup_{c \in C} \langle c, x \rangle \\\\ &= \sup_{z \in a + C} \langle z, x \rangle \\\\ &\leq \sup_{z \in B + C} \langle z, x \rangle, \end{align}

a contradiction.

$\endgroup$
2
$\begingroup$

Hmm, I have an idea for a proof which works even when $A$ isn't convex:

$A \not\subset B$ iff there is a $a \in A$ and vector $x$ such that $\langle a, x\rangle >0$ and $\langle b, x \rangle \leq 0$ for all $b \in B$ (since $B$ is a closed convex set). Let $b \in B$ and $c \in C$ maximize $\langle b + c, x \rangle$, then $\langle a + c, x \rangle > \langle b + c, x \rangle$, so there is a point in $A + C$ that isn't in $B+C$.

This implies that if $A \not\subset B$ then $A + C \not\subset B+C$. Of course, there aren't really $b,c$ which maximize that quantity above, but I believe you could make this rigorous using approximation arguments.

$\endgroup$
1
  • $\begingroup$ Just consider the suprema of the inner product with $x$ $\endgroup$ Apr 6, 2021 at 12:46

Your Answer

By clicking “Post Your Answer”, you agree to our terms of service and acknowledge you have read our privacy policy.

Not the answer you're looking for? Browse other questions tagged or ask your own question.